You are on page 1of 16

RULE 114

BAIL
Section 1. Bail Defined
CASES:
Paderanga vs. Court of Appeals
247 SCRA 741
Facts:
Petitioner Miguel Paderanga was belatedly charged as co-conspirator in the crime of multiple murder for the killing of
members of the Bucag family in Gingoog City of which, petitioner was the mayor at the time.
The trial of the case was all set to start with the issuance of an arrest warrant for petitioners apprehension, but before it
could be served on him, petitioner, through his counsel filed a motion for admission to bail with the trial court. Petitioner furnished
copies of the motion to State Prosecutor Gingoyon , to the Regional State prosecutor and the private prosecutor. The trial
proceeded to hear the application for bail, but only assistant prosecutor appeared for the prosecution and four of petitioners
counsel.
Petitioner was then confined at the Cagayan Capitol College General Hospital due to acute costochondritis, his counsel
manifested that they were submitting custody over the person of their client to the local chapter of the IBP and that, for purposes of
said hearing of his bail application, he is considered being in custody of the law. The prosecution neither supported nor opposed the
said application for bail. The trial court admitted petitioner to bail. The following day, petitioner managed to personally appear before
the clerk of court of the trial court and posted bail in the amount fixed. He was thereafter arraigned and in the trial that ensued, he
also appeared personally and attended all the scheduled court hearings of the case.
20 days after the resolution of the trial court, Prosecutor Gingoyon filed a motion for reconsideration alleging that he
received a copy of the petition for admission to bail only a day after the hearing. The trial court denied the motion for
reconsideration, and so Prosecutor Gingoyon elevated the matter the respondent Court of Appeals. The Court of Appeals annulled
the decision of the trial court in granting bail to petitioner on the ground that they were tainted with grave abuse of discretion.
Responednt court observed that at the time of petitioners application for bail, he was not yet in the custody of the law, apparently
because he filed his motion for admission to bail before he was actually arrested or had voluntarily surrendered. Respondent court
also noted that petitioner was charged with a crime punishable by reclusion perpetua, the evidence of guilt was strong as borne out
by the fact that no bail was recommended by the prosecution, for which reasons held that the grant of bail was doubly improvident.

Issue:
A.Whether or not petitioner was in the custody of the law when it applied for bail before the arrest warrant was served on
him.
B. Whether or not bail was granted with grave abuse of discretion considering that petitioner was charged with a crime
punishable by reclusion perpetua and the evidence of guilt was strong.
Ruling:
A.
Section 1 of Rule 114, as amended, defines bail as the security given for the release of a person in custody of the law,
furnished by him or a bondsman, conditioned upon his appearing before any court as required under the conditions specified in said
Rule. Its main purpose, then, is to relieve an accused from the rigors of imprisonment until his conviction and yet secure his
appearance at the trial. 10As bail is intended to obtain or secure one's provisional liberty, the same cannot be posted before custody
over him has been acquired by the judicial authorities, either by his lawful arrest or voluntary surrender. 11As this Court has put it in a
case "it would be incongruous to grant bail to one who is free."
As a paramount requisite then, only those persons who have either been arrested, detained, or other wise deprived of
their freedom will ever have occasion to seek the protective mantle extended by the right to bail. The person seeking his provisional
release under the auspices of bail need not even wait for a formal complaint or information to be filed against him as it is available to
"all persons" where the offense is bailable. The rule is, of course, subject to the condition or limitation that the applicant is in the
custody of the law.
On the other hand, a person is considered to be in the custody of the law (a) when he is arrested either by virtue of a
warrant of arrest issued pursuant to Section 6, Rule 112, or by warrantless arrest under Section 5, Rule 113 in relation to Section 7,
Rule 112 of the revised Rules on Criminal Procedure, or (b) when he has voluntarily submitted himself to the jurisdiction of the court
by surrendering to the proper authorities.
In the case of herein petitioner, it may be conceded that he had indeed filed his motion for admission to bail before he was
actually and physically placed under arrest. He may, however, at that point and in the factual ambience therefore, be considered as
being constructively and legally under custody. Thus in the likewise peculiar circumstance which attended the filing of his bail
application with the trail court, for purposes of the hearing thereof he should be deemed to have voluntarily submitted his person to
the custody of the law and, necessarily, to the jurisdiction of the trial court which thereafter granted bail as prayed for. In fact, an
arrest is made either by actual restraint of the arrestee or merely by his submission to the custody of the person making the arrest.
19
The latter mode may be exemplified by the so-called "house arrest" or, in case of military offenders, by being "confined to quarters"
or restricted to the military camp area
B.
Section 13, Article III of the Constitution lays down the rule that before conviction, all indictees shall be allowed bail,
except only those charged with offenses punishable by reclusion perpetua when the evidence of guilt is strong. In pursuance
thereof, Section 4 of Rule 114, as amended, now provides that all persons in custody shall, before conviction by a regional trial court
of an offense not punishable by death, reclusion perpetua or life imprisonment, be admitted to bail as a matter of right. The right to
bail, which may be waived considering its personal nature and which, to repeat, arises from the time one is placed in the custody of
the law, springs from the presumption of innocence accorded every accused upon whom should not be inflicted incarceration at the
outset since after trial he would be entitled to acquittal, unless his guilt be established beyond reasonable doubt.

Thus, the general rule is that prior to conviction by the regional trial court of a criminal offense, an accused is entitled to be
released on bail as a matter of right, the present exceptions thereto being the instances where the accused is charged with a capital
offense or an offense punishable by reclusion perpetua or life imprisonment and the evidence of guilt is strong. Under said general
rule, upon proper application for admission to bail, the court having custody of the accused should, as a matter of course, grant the
same after a hearing conducted to specifically determine the conditions of the bail in accordance with Section 6 (now, Section 2) of
Rule 114. On the other hand, as the grant of bail becomes a matter of judicial discretion on the part of the court under the
exceptions to the rule, a hearing, mandatory in nature and which should be summary or otherwise in the discretion of the court, is
required with the participation of both the defense and a duly notified representative of the prosecution, this time to ascertain
whether or not the evidence of guilt is strong for the provisional liberty of the applicant. Of course, the burden of proof is on the
prosecution to show that the evidence meets the required quantum.
The Court ruled that Prosecutor Abejo was authorized and could validly represent the prosecution in the hearing for bail.
He was also explicitly instructed about the position of the Regional State Prosecutors Office, to manifest to the court that the
prosecution was neither supporting nor opposing the application for bail and that they were submitting the matter to its sound
discretion. Obviously, what this meant was that the prosecution, at that particular posture of the case, was waiving the presentation
of any countervailing evidence. The prosecution was dispensing with the introduction of evidence en contra and this it did at the
proper forum and stage of the proceedings, that is during the mandatory hearing for bail and after the trial court had fully satisfied
itself that such was the position of the prosecution.
SERAPIO VS. SANDIGANBAYAN
396 SCRA 443
Facts:
Petitioner Edward Serapio was a member of the Board of Trustees and the legal counsel of the Erap Muslim Youth
Foundation. Sometime 2000, petitioner received on its behalf a donation in the amount of Php 200M through Chavit Singson.
Petitioner received the donation worth the Foundations account. In 2000, Chavit Singson publicly accused President Estrada and
his family members and friends of engaging in several illegal activities which triggered the filing with the Office of the Ombudsman
several criminal complaints against the petitioner, Joseph Estrada and his son.
On April 4, 2001, Ombudsman filed with the Sandiganbayan Informations against the former president, one of which, for
plunder. No bail was recommended for the provisional release of all the accused including the petitioner. The case was raffled to a
special division which was subsequently created by the Supreme Court. On 25 April 2001, Sandiganbayan issued a resolution
finding probable cause to justify the issuance of warrants of arrest for the accused. Arraignment was set on 27 January 2001. In
the meantime, petitioner filed with Sandiganbayan an Urgent Petition for bail, which was set for hearing on May 4, 2001.
Petitioners co-accused Jinggoy Estrada filed a motion alleging that he was entitle to bail as a matter of right.
During the hearing on May 4, 2001 on petitioners Urgent Petition for Bail, the prosecution moved for the resetting of the
arraignment of the accused earlier than the June 27 schedule. However, Sandiganbayan denied the motion of the prosecution and
issued an order declaring that the petition for bail can and should be heard BEFORE petitioners arraignment on 27 June. On June
1, Sandiganbayan issued a resolution requiring the attendance of petitioner as well as all the other accused during the hearing on
the petitioner for bail considering that under Section 8, Rule 115 of the Revised Rules of Court, whatever evidence adduced during
the hearing shall be considered automatically reproduced at the trial.
The people insist that arraignment is necessary before bail hearings may be commenced because it is only upon
arraignment that the issues are joined. The people further stress the it is only when an accused pleads not guilty may he filed a
petition for bail and if he pleads guilty, then there would be no need for him to file said petition. It is also the contention of the people
that it is only during arraignment that the accused is informed of the precise charge against him. He must then be arraign first prior
to bail hearings to prevent him from late on assailing the validity of the bail hearings on the ground that he was not properly informed
of the charge considering that under section 8 of Rule 114, evidence presented during bail hearings are reproduce in the trial.
Arraignment before bail hearings also diminished the possibility of accuseds flight since trial in absentia may be had only if an
accused escapes after he has been arraigned.
However, the bail hearing again did not proceed because the petitioner filed with the information a motion to quash the
amended information on the grounds that as against him, the amended information does not allege a combination of series of over
or criminal acts constitutive of plunder. According to the prosecution, the motion to quash the amended information was antithetical
to his petition for bail.
Petitioner also prays for the issuance of habeas corpus.
Issues:
(a)

W/N petitioner should first be arraigned before hearings of his petition for bail may be conducted.

(b)

W/N petitioner may file a motion to quash the amended Information during the pendency of his petition for bail.

(c)

W/N a joint hearing of petition for bail for all the accused is mandatory

(d)

W/N petitioner should instead be released through a writ of habeas corpus.

Ruling:
(a)
Although the petitioner was already arraigned, no plea has yet been entered thereby rendering the issue of whether an
arraignment is necessary before the conduct of bail hearings in the petitioners case moot. Nonetheless, the court held that
arraignment of an accused is not a pre-requisite to the conduct of hearings on his petition for bail. A person is allowed to petition for
bail as soon as he is deprived of his of his liberty by virtue of his arrest or voluntary surrender.
In Lavides vs. CA, the court ruled that in cases where it is authorized, bail should be granted before arraignment
otherwise the accused may be precluded from filing a motion to quash. However, this pronouncement should not be taken to mean
that the hearing on a petition for bail should at all times precede arraignment, because the rule is that a person deprived of his
liberty by virtue of his arrest or voluntary surrender may apply for bail as soon as he is deprived of liberty even before a complaint or
information is filed against him. The case of Lavides must be understood in light of the fact that the accused in said case filed a
petition for bail as well as a motion to quash. Hence, in that case, the court held that to condition the grant of bail to an accused on

his arraignment would be to place him in a position where he had to choose between filing a motion to quash and thus delay his
petition for bail and forgoing the filing of the motion to quash so that he can be arraign at once ad therefore be released on bail.
Such would undermine the constitutional right of the accused.
When a bail is matter of right, an accused may apply for and be granted bail even prior to arraignment. The Lavides case
also implies that an application for bail in a case involving an offense punishable by reclusion perpetua to death may also be heard
even before an accused is arraigned. Sandiganbayan therefore committed grave abuse of discretion amounting to excess of
jurisdiction in ordering the arraignment of petitioner before proceeding with the hearing of his petition for bail.
(b)
Court dins no inconsistency exists between an application of an accused for bail and his filing of a motion to quash. Bail,
is the security given for the release of the person in custody of the law. A motion to quash on the other hand is a mode by which an
accused assails the validity of a criminal complaint filed against him for insufficiency on its fact in posit of law. These tow relied have
objectives which are not necessarily antithetical to each other. However, it is true that if a motion to quash a criminal complaint or
information on the ground that the same does not charge any offense is granted and the case is dismissed and the accused is
ordered released, the petition for bail of an accused may become moot and academic.
(c)
Petitioner argues that a joint bail hearing would negate his right to have his petition for bail resolved in a summary
proceeding since said hearing might be converted into a full blown trial. Prosecution on the other hand claims that joint hearings will
save the court form having to hear the same witnesses and the parties from presenting the same evidences. There is no provision
in the Rules of Court governing the hearings of two or more petitioner for bail filed by different accused or that a petition for bail of an
accused be heard simultaneously with the trial of the case against the other accused. The matter should be addressed to the sound
discretion of the trial court. In the exercise of its discretion, the Sandiganbayan must take into account not only the convenience of
the state, including the prosecution but also that of the petitioner and the witnesses.
In the case of Ocampo vs. Bernabe, the court ruled that in a petition or bail hearing, the court is to conduct only a
summary hearing, meaning such brief and speedy method of receiving and considering the evidence of guilt as is practicable and
consistent with the purpose of the hearing which is early to determine the weight of evidence for purposes of bail. The court does
not try the merits or enter into the inquiry as to the weight that ought to be given to the evidence against the accused, nor will it
speculate on the outcome of the trial or on what further such evidence as has reference to substantial matters. In the case at bar,
the case against former President Estrada is an entirely different matter. For, with the participation of the former president in the
hearing of petitioners petition for bail, the proceeding assumes completely different dimension. The proceeding will no longer be
summary since the proceedings will be full blown which is antithetical to the nature of a bail hearing. The joinder of the petitioners
bail will be prejudicial to the petitioner as it will unduly delay the determination of the issue of the right of petitioner to obtain
provisional liberty and seek relief from his court. The Sandiganbayn again committed a grave abuse of discretion in ordering a
simultaneous hearing of petitioners petition for bail with the trial of the case against former president.

(d)
In the case at bar, bail is not matter of rights since the accused is charged with a capital offense, but discretionary upon
the court. Under Section 8 of rule 114, there must be a showing that the evidence of guilt against a person charged with a capital
offense is not strong for the court to grant him bail., thus, upon an application for bail, by the person charged with a capital offense,
a hearing must be conducted where the prosecution has the burden of showing that the evidence of guilt against an accused is
strong. When the evidence of guilt is strong, bail becomes a matter of right, which is not so in the case at bar.
In exceptional cases, habeas corpus may be granted ny the courts even when the person concerned is detained pursuant
to a valid arrest or his voluntary surrender. The writ may be issued where the deprivation of liberty while initially valid under the lad
had not later become invalid. However, there is no basis fir the issuance of the writ in the case at bar. The general rule is that the
writ does not lie where the person alleged to be restrained of his liberty is in the custody of an officer under process issued by a
court which had jurisdiction to issued the same applied, because petitioner is under detention pursuant to the order of arrest.
Petitioner in fact voluntarily surrendered himself to the authorities.
Commendador et.al vs. De Villa
G.R. No. 93177 August 2, 1991
CRUZ, J.:p
Topic: Fundamental Principles
FACTS:
The private respondents are officers of the Armed Forces of the Philippines facing prosecution for their alleged participation in the
failed coup d' etat that took place on December 1 to 9, 1989.
The charges against them are violation of Articles of War (AW) 67 (Mutiny), AW 96 (Conduct Unbecoming an Officer and a
Gentleman) and AW 94 (Various Crimes) in relation to Article 248 of the Revised Penal Code (Murder).
Ltc Jacinto Ligot (private respondent) applied for bail, but the application was denied by GCM No.14. He thereupon filed with the
Regional Trial Court of Quezon City a petition for certiorari and mandamus with prayer for provisional liberty and a writ of preliminary
injunction which consequently issued an order granting provisional liberty to Ligot.
The trial court rendered judgment inter alia:
Xxx Declaring, that Section 13, Article III of the Constitution granting the right to bail to all persons with the defined
exception is applicable and covers all military men facing court-martial proceedings. xxx
However, Solicitor general contends that military men are exempt from constitutional coverage on the right to bail.
ISSUE:
Do private respondents have the right to bail?
RULING:
None. Supreme Court find that the right to bail invoked by the private respondents has traditionally not been recognized and is not
available in the military, as an exception to the general rule embodied in the Bill of Rights. This much was suggested in Arula, where
we observed that "the right to a speedy trial is given more emphasis in the military where the right to bail does not exist.
The argument that denial from the military of the right to bail would violate the equal protection clause is not acceptable.

This guaranty requires equal treatment only of persons or things similarly situated and does not apply where the subject of the
treatment is substantially different from others. The accused officers can complain if they are denied bail and other members of the
military are not. But they cannot say they have been discriminated against because they are not allowed the same right that is
extended to civilians.
TRILLANES IV vs HON PIMENTEL
FACTS: Petitioner Trillanes, who is charged and detained for staging a coup d'etat referred to as the Oakwood Incident asked the
Court that he be allowed to attend all official functions of the Senate. While he was in detention, he ran and won a seat in the
Senate. He filed with the RTC an "Omnibus Motion for Leave of Court to be Allowed to Attend Senate Sessions and Related
Requests". The trial court denied all the requests in the Omnibus Motion as well as the motion for reconsideration.
Petitioner then filed a petition for certiorari to set aside the two Orders and for prohibition and mandamus to (i) enjoin respondents
from banning the Senate staff, resource persons and guests from meeting with him or transacting business with him in his capacity
as Senator; and (ii) direct respondents to allow him access to the Senate staff, resource persons and guests and permit him to
attend all sessions and official functions of the Senate. He stated that he should be allowed bail because he still enjoys the
presumption of innocence since unlike Jalosjos he has not been convicted of the crime charged against him. Further, Jalosjos was
charged with statutory rape and acts of lasciviousness, crimes involving moral turpitude, while the petitioner is charged with coup d
etat which is commonly regarded as a political offense. He further alleged that unlike Jalosjos who attempted to evade trial, he is not
a flight risk since he voluntarily surrendered to the proper authorities and such can be proven by the numerous times he was
allowed to travel outside his place of detention.
ISSUE: Whether or not the accused should be allowed bail to attend to his official functions in senate since unlike Jalosjos he has
not been convicted and still enjoys the presumption of innocence
HELD: The petitioners application for bail and for release on recognizance was denied. The constitution provides that:
All persons, except those charged with offenses punishable by reclusion perpetua when evidence of guilt is strong, shall, before
conviction, be bailable by sufficient sureties or be released on recognizance as may be provided by law. The right to bail shall not
be impaired even when the privilege of the writ of habeas corpus is suspended. Excessive bail shall not be required.
Within the class of offenses covered by the stated range of imposable penalties, there is clearly no distinction as to the political
complexion of or moral turpitude involved in the crime charged. The determination that the evidence of guilt is strong, whether
ascertained in a hearing of an application for bail or imported from a trial court's judgment of conviction justifies the detention of an
accused as a valid curtailment of his right to provisional liberty. This accentuates the proviso that the denial of the right to bail in
such cases is "regardless of the stage of the criminal action." Such justification for confinement with its underlying rationale of public
self-defense applies equally to detention prisoners like petitioner or convicted prisoners-appellants like Jalosjos.
In People v. Hon. Maceda, the court stated that when a person indicted for an offense is arrested, he is deemed placed under the
custody of the law. He is placed in actual restraint of liberty in jail so that he may be bound to answer for the commission of the
offense. He must be detained in jail during the pendency of the case against him, unless he is authorized by the court to be
released on bail or on recognizance. Let it be stressed that all prisoners whether under preventive detention or serving final
sentence can not practice their profession nor engage in any business or occupation, or hold office, elective or appointive, while in
detention. This is a necessary consequence of arrest and detention. These inherent limitations, however, must be taken into account
only to the extent that confinement restrains the power of locomotion or actual physical movement. It bears noting that in Jalosjos,
which was decided en banc one month after Maceda, the Court recognized that the accused could somehow accomplish legislative
results. The trial court thus correctly concluded that the presumption of innocence does not carry with it the full enjoyment of civil
and political rights.
Petitioner is similarly situated with Jalosjos with respect to the application of the presumption of innocence during the period material
to the resolution of their respective motions. The Court in Jalosjos did not mention that the presumption of innocence no longer
operates in favor of the accused pending the review on appeal of the judgment of conviction. The rule stands that until a
promulgation of final conviction is made, the constitutional mandate of presumption of innocence prevails. His contention that he is
not a flight risk for voluntarily surrendering to the proper authorities is contravened by subsequent events. He went past security
detail and proceeded from the courtroom to a posh hotel to issue certain statements. The account, dubbed this time as the "Manila
Pen Incident," proves that petitioner's argument bites the dust. The risk that he would escape ceased to be neither remote nor nil
as, in fact, the cause for foreboding became real. Moreover, circumstances indicating probability of flight find relevance as a factor in
ascertaining the reasonable amount of bail and in canceling a discretionary grant of bail. In cases involving non-bailable offenses,
what is controlling is the determination of whether the evidence of guilt is strong. Once it is established that it is so, bail shall be
denied as it is neither a matter of right nor of discretion. Petitioner cannot find solace in Montano v. Ocampo to buttress his plea for
leeway because unlike petitioner, then Senator Justiniano Montano was able to rebut the strong evidence for the prosecution.
Notatu dignum is this Court's pronouncement therein that "if denial of bail is authorized in capital cases, it is only on the theory that
the proof being strong, the defendant would flee, if he has the opportunity, rather than face the verdict of the jury.
Jaloslos succinctly expounds that allowing accused-appellant to attend congressional sessions and committee meetings for five (5)
days or more in a week will virtually make him a free man with all the privileges appurtenant to his position. Such an aberrant
situation not only elevates accused-appellant's status to that of a special class, it also would be a mockery of the purposes of the
correction system. WHEREFORE, the petition is DISMISSED.
SECTION 2-9. CASES
Manotoc vs. CA (May 30, 1986)

FACTS:
Ricardo Manotoc Jr. was one of the two principal stockholders of Trans-Insular Management Inc. and the Manotoc Securities Inc.
(stock brokerage house). He was in US for a certain time, went home to file a petition with SEC for appointment of a management
committee for both businesses. Such was granted. However, pending disposition of a case filed with SEC, the latter requested the
Commissioner of Immigration not to clear him for departure. Consequently, a memorandum to this effect was issued.
There was a torrens title submitted to and accepted by Manotoc Securities Inc which was suspected to be fake. 6 of its clients filed
separate criminal complaints against the petitioner and Leveriza, President and VP respectively. He was charged with estafa and
was allowed by the Court to post bail.
Petitioner filed before each trial court motion for permission to leave the country stating his desire to go to US relative to his
business transactions and opportunities. Such was opposed by the prosecution and was also denied by the judges. He filed
petition for certiorari with CA seeking to annul the prior orders and the SEC communication request denying his leave to travel
abroad.
According to the petitioner, having been admitted to bail as a matter of right, neither the courts that granted him bail nor SEC, which
has no jurisdiction over his liberty, could prevent him from exercising his constitutional right to travel.

ISSUE: WON petitioners constitutional right to travel was violated.


HELD: NO.
The court has power to prohibit person admitted to bail from leaving the country because this is a necessary consequence of the
nature and function of a bail bond. The condition imposed upon petitioner to make himself available at all times whenever
the court requires his presence operates as a valid restriction on his constitutional right to travel. In case he will be allowed
to leave the country without sufficient reason, he may be placed beyond the reach of courts.
Furthermore, petitioner failed to satisfy trial court and CA of the urgency of his travel, duration thereof, as well as consent of his
surety to the proposed travel. He was not able to show the necessity of his travel abroad. He never indicated that no other person
in his behalf could undertake such business transaction.
Article 3 Sec6: The liberty of abode and of changing the same shall not be impaired except upon lawful order of the
court. According to SC, the order of trial court in releasing petitioner on bail constitutes such lawful order as contemplated by the
provision on right to travel.

G.R. No. 94284 April 8, 1991


RICARDO
C.
SILVERIO, petitioner,
vs.
THE COURT OF APPEALS, HON. BENIGNO G. GAVIOLA, as Judge of the Regional Trial Court of Cebu City, Branch IX, and
PEOPLE OF THE PHILIPPINES
Facts:
This is a Petition for Review on Certiorari under Rule 45 of the Rules of Court praying that the Decision of respondent Court of
Appeals in CA-G.R. SP No. 15827, entitled "Ricardo C. Silverio vs. Hon. Benigno C. Gaviola, etc., et al.," dated 31 January 1990, as
well as the Resolution of 29 June 1990 denying reconsideration, be set aside.
On 14 October 1985, Petitioner was charged with violation of Section 20 (4) of the Revised Securities Act in Criminal Case No.
CBU-6304 of the Regional Trial Court of Cebu. In due time, he posted bail for his provisional liberty.
On 26 January 1988, or more than two (2) years after the filing of the Information, respondent People of the Philippines filed an
Urgent ex parte Motion to cancel the passport of and to issue a hold-departure Order against accused-petitioner on the ground that
he had gone abroad several times without the necessary Court approval resulting in postponements of the arraignment and
scheduled hearings.
Overruling opposition, the Regional Trial Court, on 4 April 1988, issued an Order directing the Department of Foreign Affairs to
cancel Petitioner's passport or to deny his application therefor, and the Commission on Immigration to prevent Petitioner from
leaving the country. This order was based primarily on the Trial Court's finding that since the filing of the Information on 14 October
1985, "the accused has not yet been arraigned because he has never appeared in Court on the dates scheduled for his arraignment
and there is evidence to show that accused Ricardo C. Silverio, Sr. has left the country and has gone abroad without the knowledge
and permission of this Court" (Rollo, p. 45). Petitioner's Motion for Reconsideration was denied on 28 July 1988.
Petitioner's Certiorari Petition before the Court of Appeals met a similar fate on 31 January 1990. Hence, this Petition for Review
filed on 30 July 1990.
ISSUE:
Whether or not the right to travel can be impaired upon lawful order of the Court, even on grounds other than the "interest of national
security, public safety or public health."
RULING:
Yes, The bail bond he had posted had been cancelled and Warrants of Arrest had been issued against him by reason, in both
instances, of his failure to appear at scheduled arraignments. Warrants of Arrest having been issued against him for violation of the
conditions of his bail bond, he should be taken into custody. "Bail is the security given for the release of a person in custody of the
law, furnished by him or a bondsman, conditioned upon his appearance before any court when so required by the Court or the Rules
Article III, Section 6 of the 1987 Constitution should by no means be construed as delimiting the inherent power of the Courts to use
all means necessary to carry their orders into effect in criminal cases pending before them. When by law jurisdiction is conferred on
a Court or judicial officer, all auxillary writs, process and other means necessary to carry it into effect may be employed by such
Court or officer
Petitioner is facing a criminal charge. He has posted bail but has violated the conditions thereof by failing to appear before the Court
when required. Warrants for his arrest have been issued. Those orders and processes would be rendered nugatory if an accused
were to be allowed to leave or to remain, at his pleasure, outside the territorial confines of the country. Holding an accused in a
criminal case within the reach of the Courts by preventing his departure from the Philippines must be considered as a valid
restriction on his right to travel so that he may be dealt with in accordance with law. The offended party in any criminal proceeding is
the People of the Philippines . It is to their best interest that criminal prosecutions should run their course and proceed to finality
without undue delay, with an accused holding himself amenable at all times to Court Orders and processes.
The judgment under review is hereby AFFIRMED.

People v. Intermediate Appellate Court


147 SCRA 219
FACTS:
As a result of a shooting incident the accused was charged with two counts of assault upon an agent in authority and murder with
the use of an illegally possessed firearm. The trial court denied the petition for bail of the accused on the ground that the crimes
charged were capital offenses and the evidence of guilt was strong. The Intermediate Appellate Court granted bail to the accused on
the ground that the crimes committed involved homicide and not murder.
SC RULING:
To determine whether the offense charged is capital, the criterion is the penalty regardless of the attendant circumstances.
Otherwise, evidence of the aggravating and mitigating circumstance will have to be considered. There will be a complete trial. This
will defeat the purpose of bail.

EDWARD S. SERAPIO, petitioner, vs. HONORABLE SANDIGANBAYAN and PEOPLE OF THE PHILIPPINES
FACTS;
Before the Court are two petitions for certiorari filed by petitioner Edward Serapio, assailing the resolutions of the Third Division of
the Sandiganbayan denying his petition for bail.
The records show that petitioner was a member of the Board of Trustees and the Legal Counsel of the Erap Muslim Youth
Foundation, a non-stock, non-profit foundation established in February 2000 ostensibly for the purpose of providing educational

opportunities for the poor and underprivileged but deserving Muslim youth and students, and support to research and advance
studies of young Muslim educators and scientists.
Sometime in April 2000, petitioner, as trustee of the Foundation, received on its behalf a donation in the amount of Two
Hundred Million Pesos (P200 Million) from Ilocos Sur Governor Luis Chavit Singson through the latters assistant Mrs. Yolanda
Ricaforte. Petitioner received the donation and turned over the said amount to the Foundations treasurer who later deposited it in
the Foundations account with the Equitable PCI Bank.
. During the hearing on May 4, 2001 on petitioners Urgent Petition for Bail, the prosecution moved for the resetting of the
arraignment of the accused earlier than the June 27, 2001 schedule. However, the Sandiganbayan denied the motion of the
prosecution and issued an order declaring that the petition for bail can and should be heard before petitioners arraignment on June
27, 2001 and even before the other accused in Criminal Case No. 26558 filed their respective petitions for bail. Accordingly, the
Sandiganbayan set the hearing for the reception of evidence on petitioners petition for bail on May 21 to 25, 2001.

ISSUE:
Whether or not petitioner should first be arraigned before hearings of his petition for bail may be conducted.
HELD:
The arraignment of an accused is not a prerequisite to the conduct of hearings on his petition for bail. A person is allowed to petition
for bail as soon as he is deprived of his liberty by virtue of his arrest or voluntary surrender (Mendoza vs. CFI of Quezon, 51 SCAD
369). an accused need not wait for his arraignment before filing a petition for bail. In Lavides vs. CA, 324 SCRA 321, it was held that
in cases where it is authorized, bail should be granted before arraignment, otherwise the accused may be precluded from filing a
motion to quash. This pronouncement should be understood in the light of the fact that the accused in said case filed a petition for
bail as well as a motion to quash the informations filed against him. It was explained that to condition the grant of bail to an accused
on his arraignment would be to place him in a position where he has to choose between: (1) filing a motion to quash and thus delay
his release on bail because until his motion to quash can be resolved, his arraignment cannot be held; and (2) foregoing the filing of
a motion to quash so that he can be arraigned at once and thereafter be released on bail. This would undermine his constitutional
right not to be put on trial except upon a valid complaint or information sufficient to charge him with a crime and his right to bail. It is
therefore not necessary that an accused be first arraigned before the conduct of hearings on his application for bail. For when bail is
a matter of right, an accused may apply for and be granted bail even prior to arraignment .

ROLITO GO y TAMBUNTING vs. COURT OF APPEALS


206 SCRA 138
FACTS: An information was filed charging herein petitioner Rolito Go for murder before the Regional Trial Court of Metro Manila.
Petitioner voluntarily presented himself together with his two lawyers to the police upon obtaining knowledge of being hunted by the
latter. However, he was immediately detained and denied his right of a preliminary investigation unless he executes and sings a
waiver of the provisions of Article 125 of the Revised Penal Code. Upon omnibus motion for immediate release on recognizance or
on bail and proper preliminary investigation on the ground that his warrantless arrest was unlawful and no preliminary investigation
was conducted before the information was filed, which is violative of his rights, the same was granted but later on reversed by the
lower court and affirmed by the Court of Appeals. The appellate court in sustaining the decision of the lower court held that
petitioners warrantless arrest was valid in view of the fact that the offense was committed, the petitioner was clearly identified and
there exists valid information for murder filed against petitioner.
Hence, the petitioner filed this present petition for review on certiorari before the Supreme Court.
ISSUE/S: The
issues
assailed
in
the
case
at
bar
are
the
following:
1.
whether
or
not
the
warrantless
arrest
of
herein
petitioner
was
lawful,
and
2. whether or not petitioner waived his right to preliminary investigation.
RULING: The general rule on arrest provides that the same is legitimate if effected with a valid warrant. However, there are
instances specifically enumerated under the law when a warrantless arrest may be considered lawful. Despite that, the warrantless
arrest of herein petitioner Rolito Go does not fall within the terms of said rule. The police were not present at the time of the
commission of the offense, neither do they have personal knowledge on the crime to be committed or has been committed not to
mention the fact that petitioner was not a prisoner who has escaped from the penal institution. In view of the above, the allegation of
the prosecution that petitioner needs to sign a waiver of the provisions of Article 125 of the Revised Penal Code before a preliminary
investigation may be conducted is baseless. In this connection, petitioner has all the right to ask for a preliminary investigation to
determine whether is probable cause that a crime has been committed and that petitioner is probably guilty thereof as well as to
prevent him from the hassles, anxiety and aggravation brought by a criminal proceeding. This reason of the accused is substantial,
which he should not be deprived of.
On the other hand, petitioner did not waive his right to have a preliminary investigation contrary to the prosecutors claim. The right
to preliminary investigation is deemed waived when the accused fails to invoke it before or at the time of entering a pleas at
arraignment. The facts of the case show that petitioner insisted on his right to preliminary investigation before his arraignment and
he, through his counsel denied answering questions before the court unless they were afforded the proper preliminary investigation.
For the above reasons, the petition was granted and the ruling of the appellate court was set aside and nullified. The Supreme Court
however, contrary to petitioners allegation, declared that failure to accord the right to preliminary investigation did not impair the
validity of the information charging the latter of the crime of murder.

G.R. No. 148571

September 24, 2002

GOVERNMENT OF THE UNITED STATES OF AMERICA,


Represented by the Philippine Department of Justice, petitioner,
vs.
HON. GUILLERMO PURGANAN, Presiding Judge Regional Trial Court of Manila and
MARC JIMENEZ a.k.a. MARCIO BATACAN CRESPO, respondent

Facts:
Pursuant to the existing RP-US Extradition Treaty, the United States Government requested the extradition of Mark B.
Jimenez, also known as Mario Batacan Crespo. Upon receipt of the request, the secretary of foreign affairs (SFA) transmitted them
to the secretary of justice (SOJ) for appropriate action.
The Government of the United States of America, represented by the Philippine DOJ, filed with the Regional Trial Court
the appropriate Petition for Extradition.

Before the Regional Trial Court could act on the Petition, Respondent Jimenez filed before it an "Urgent Manifestation/ExParte Motion," which prayed that petitioners application for an arrest warrant be set for hearing. Jimenez sought an alternative
prayer: that in case a warrant should issue, he be allowed to post bail in the amount of P100, 000.
Thereafter, the Regional Trial Court issued its questioned order, directing the issuance of a warrant for his arrest and fixing
bail for his temporary liberty at one million pesos in cash. After he had surrendered his passport and posted the required cash bond,
Jimenez was granted provisional liberty.
Hence, this Petition. Essentially, the Petition prays for the lifting of the bail Order, the cancellation of the bond, and the
taking of Jimenez into legal custody.

Issue:
Whether he is entitled to bail and to provisional liberty while the extradition proceedings are pending.

Ruling:
Article III, Section 13 of the Constitution, is worded as follows:
Art. III, Sec. 13. All persons, except those charged with offenses punishable by reclusion perpetua when evidence of guilt is
strong, shall, before conviction, be bailable by sufficient sureties, or be released on recognizance as may be provided by law. The
right to bail shall not be impaired even when the privilege of the writ of habeas corpus is suspended. Excessive bail shall not be
required."
As suggested by the use of the word "conviction," the constitutional provision on bail quoted above, as well as Section 4
of Rule 114 of the Rules of Court, applies only when a person has been arrested and detained for violation of Philippine criminal
laws. It does not apply to extradition proceedings, because extradition courts do not render judgments of conviction or acquittal.
Moreover, the constitutional right to bail "flows from the presumption of innocence in favor of every accused who should not
be subjected to the loss of freedom as thereafter he would be entitled to acquittal, unless his guilt be proved beyond reasonable
doubt." It follows that the constitutional provision on bail will not apply to a case like extradition, where the presumption of innocence
is not at issue.
That the offenses for which Jimenez is sought to be extradited are bailable in the United States is not an argument to
grant him one in the present case. To stress, extradition proceedings are separate and distinct from the trial for the offenses for
which he is charged. He should apply for bail before the courts trying the criminal cases against him, not before the extradition court.
There is an exception to the No Bail Rule to best serve the ends of justice. After a potential extraditee has been arrested
or placed under the custody of the law, bail may be applied for and granted as an exception, only upon a clear and convincing
showing (1) that, once granted bail, the applicant will not be a flight risk or a danger to the community; and (2) that there exist
special, humanitarian and compelling circumstances including, as a matter of reciprocity, those cited by the highest court in the
requesting state when it grants provisional liberty in extradition cases therein.
Jimenez claims that he is not a flight risk. To support this claim, he stresses that he learned of the extradition request in June
1999; yet, he has not fled the country. True, he has not actually fled during the preliminary stages of the request for his extradition.
Yet, this fact cannot be taken to mean that he will not flee as the process moves forward to its conclusion, as he hears the footsteps
of the requesting government inching closer and closer. That he has not yet fled from the Philippines cannot be taken to mean that
he will stand his ground and still be within reach of our government if and when it matters; that is, upon the resolution of the Petition
for Extradition.
In extradition cases, bail is not a matter of right; it is subject to judicial discretion in the context of the peculiar
facts of each case.
WHEREFORE, the Petition is GRANTED. The assailed RTC Order dated May 23, 2001 is hereby declared NULL and
VOID, while the challenged Order dated July 3, 2001 is SET ASIDE insofar as it granted bail to Respondent Mark Jimenez. The bail
bond posted by private respondent is CANCELLED. The Regional Trial Court of Manila is directed to conduct the extradition
proceedings before it, with all deliberate speed pursuant to the spirit and the letter of our Extradition Treaty with the United States as
well as our Extradition Law. No costs.
SO ORDERED.
GR. no. 153675 April 19 2007
Gov. of Hong ong vs Olalia
Facts :
On January 30, 1995, the Republic of the Philippines and the then British Crown Colony of Hong Kong signed an extradition treaty
which took effect on June 20, 1997.
On July 1, 1997, Hong Kong reverted back to the Peoples Republic of China and became the Hong Kong Special Administrative
Region.
Muoz was charged before the Hong Kong Court with three (3) counts of the offense of "accepting an advantage as agent," in
violation of Section 9 (1) (a) of the Prevention of Bribery Ordinance, Cap. 201 of Hong Kong. He also faces seven (7) counts of the
offense of conspiracy to defraud, penalized by the common law of Hong Kong. On August 23, 1997 and October 25, 1999, warrants
of arrest were issued against him. If convicted, he faces a jail term of seven (7) to fourteen (14) years for each charge.
On September 13, 1999, the DOJ received from the Hong Kong Department of Justice a request for the provisional arrest of private
respondent. The DOJ then forwarded the request to the National Bureau of Investigation (NBI) which, in turn, filed with the RTC of
Manila, Branch 19 an application for the provisional arrest of private respondent.
On September 23, 1999, the RTC, Branch 19, Manila issued an Order of Arrest against private respondent. That same day, the NBI
agents arrested and detained him.

On October 14, 1999, private respondent filed with the Court of Appeals a petition for certiorari, prohibition and mandamus with
application for preliminary mandatory injunction and/or writ of habeas corpus questioning the validity of the Order of Arrest.
On November 9, 1999, the Court of Appeals rendered its Decision declaring the Order of Arrest void.
On November 12, 1999, the DOJ filed with this Court a petition for review on certiorari, docketed as G.R. No. 140520, praying that
the Decision of the Court of Appeals be reversed.
On December 18, 2000, this Court rendered a Decision granting the petition of the DOJ and sustaining the validity of the Order of
Arrest against private respondent. The Decision became final and executory on April 10, 2001.
Meanwhile, as early as November 22, 1999, petitioner Hong Kong Special Administrative Region filed with the RTC of Manila a
petition for the extradition of private respondent, docketed as Civil Case No. 99-95733, raffled off to Branch 10, presided by Judge
Ricardo Bernardo, Jr. Judge Bernado denied Munoz petition for bail.
On October 22, 2001, Judge Bernardo, Jr. inhibited himself from further hearing Civil Case No. 99-95733. It was then raffled off to
Branch 8 presided by judge Olalia.
On October 30, 2001, Judge Olalia granted the petition for bail.
On December 21, 2001, the government of Hongkong represented by the DOJ filed an urgent motion to vacate the order on the
petition for bail, but it was denied by respondent judge in his Order dated April 10, 2002.
Issue:
Is bail available in a extradition case?
Ruling:
The Philippines, along with the other members of the family of nations, committed to uphold the fundamental human rights as well
as value the worth and dignity of every person. This commitment is enshrined in Section II, Article II of our Constitution which
provides: "The State values the dignity of every human person and guarantees full respect for human rights." The Philippines,
therefore, has the responsibility of protecting and promoting the right of every person to liberty and due process, ensuring that those
detained or arrested can participate in the proceedings before a court, to enable it to decide without delay on the legality of the
detention and order their release if justified. In other words, the Philippine authorities are under obligation to make available to every
person under detention such remedies which safeguard their fundamental right to liberty. These remedies include the right to be
admitted to bail. While this Court in Purganan limited the exercise of the right to bail to criminal proceedings, however, in light of the
various international treaties giving recognition and protection to human rights, particularly the right to life and liberty, a
reexamination of this Courts ruling in Purganan is in order.
First, we note that the exercise of the States power to deprive an individual of his liberty is not necessarily limited to criminal
proceedings. Respondents in administrative proceedings, such as deportation and quarantine,4 have likewise been detained.
Second, to limit bail to criminal proceedings would be to close our eyes to our jurisprudential history. Philippine jurisprudence has
not limited the exercise of the right to bail to criminal proceedings only. This Court has admitted to bail persons who are not involved
in criminal proceedings. In fact, bail has been allowed in this jurisdiction to persons in detention during the pendency of
administrative proceedings, taking into cognizance the obligation of the Philippines under international conventions to uphold human
rights.
The 1909 case of US v. Go-Sioco5 is illustrative. In this case, a Chinese facing deportation for failure to secure the necessary
certificate of registration was granted bail pending his appeal. After noting that the prospective deportee had committed no crime,
the Court opined that "To refuse him bail is to treat him as a person who has committed the most serious crime known to law;" and
that while deportation is not a criminal proceeding, some of the machinery used "is the machinery of criminal law." Thus, the
provisions relating to bail was applied to deportation proceedings.
In Mejoff v. Director of Prisons6 and Chirskoff v. Commission of Immigration,7 this Court ruled that foreign nationals against whom no
formal criminal charges have been filed may be released on bail pending the finality of an order of deportation. As previously stated,
the Court in Mejoff relied upon the Universal declaration of Human Rights in sustaining the detainees right to bail.
If bail can be granted in deportation cases, we see no justification why it should not also be allowed in extradition cases. Likewise,
considering that the Universal Declaration of Human Rights applies to deportation cases, there is no reason why it cannot be
invoked in extradition cases. After all, both are administrative proceedings where the innocence or guilt of the person detained is not
in issue.
Clearly, the right of a prospective extraditee to apply for bail in this jurisdiction must be viewed in the light of the various treaty
obligations of the Philippines concerning respect for the promotion and protection of human rights. Under these treaties, the
presumption lies in favor of human liberty. Thus, the Philippines should see to it that the right to liberty of every individual is not
impaired.
Section 2(a) of Presidential Decree (P.D.) No. 1069 (The Philippine Extradition Law) defines "extradition" as "the removal of an
accused from the Philippines with the object of placing him at the disposal of foreign authorities to enable the requesting state or
government to hold him in connection with any criminal investigation directed against him or the execution of a penalty imposed on
him under the penal or criminal law of the requesting state or government."
Extradition has thus been characterized as the right of a foreign power, created by treaty, to demand the surrender of one accused
or convicted of a crime within its territorial jurisdiction, and the correlative duty of the other state to surrender him to the demanding
state.8 It is not a criminal proceeding.9 Even if the potential extraditee is a criminal, an extradition proceeding is not by its nature
criminal, for it is not punishment for a crime, even though such punishment may follow extradition. 10 It is sui generis, tracing its
existence wholly to treaty obligations between different nations. 11 It is not a trial to determine the guilt or innocence of the
potential extraditee.12 Nor is it a full-blown civil action, but one that is merely administrative in character.13 Its object is to prevent
the escape of a person accused or convicted of a crime and to secure his return to the state from which he fled, for the purpose of
trial or punishment.14
But while extradition is not a criminal proceeding, it is characterized by the following: (a) it entails a deprivation of liberty on the part
of the potential extraditee and (b) the means employed to attain the purpose of extradition is also "the machinery of criminal
law." This is shown by Section 6 of P.D. No. 1069 (The Philippine Extradition Law) which mandates the " immediate arrest and
temporary detention of the accused" if such "will best serve the interest of justice." We further note that Section 20 allows the
requesting state "in case of urgency" to ask for the "provisional arrest of the accused, pending receipt of the request for
extradition;" and that release from provisional arrest "shall not prejudice re-arrest and extradition of the accused if a request for
extradition is received subsequently."
A potential extraditee may be subjected to arrest, to a prolonged restraint of liberty, and forced to transfer to the
demanding state following the proceedings. "Temporary detention" may be a necessary step in the process of extradition, but
the length of time of the detention should be reasonable.
The accused had been detained for over two (2) years without having been convicted of any crime. By any standard, such an
extended period of detention is a serious deprivation of his fundamental right to liberty. In fact, it was this prolonged deprivation of
liberty which prompted the extradition court to grant him bail.
While our extradition law does not provide for the grant of bail to an extraditee, however, there is no provision prohibiting him or her
from filing a motion for bail, a right to due process under the Constitution.
The applicable standard of due process, however, should not be the same as that in criminal proceedings. In the latter, the standard
of due process is premised on the presumption of innocence of the accused. As Purganan correctly points out, it is from this major
premise that the ancillary presumption in favor of admitting to bail arises. Bearing in mind the purpose of extradition proceedings,
the premise behind the issuance of the arrest warrant and the "temporary detention" is the possibility of flight of the potential
extraditee. This is based on the assumption that such extraditee is a fugitive from justice. 15 Given the foregoing, the prospective
extraditee thus bears the onus probandi of showing that he or she is not a flight risk and should be granted bail.
The time-honored principle of pacta sunt servanda demands that the Philippines honor its obligations under the Extradition Treaty it
entered into with the Hong Kong Special Administrative Region. Failure to comply with these obligations is a setback in our foreign
relations and defeats the purpose of extradition. However, it does not necessarily mean that in keeping with its treaty obligations, the
Philippines should diminish a potential extraditees rights to life, liberty, and due process. More so, where these rights are
guaranteed, not only by our Constitution, but also by international conventions, to which the Philippines is a party. We should not,
therefore, deprive an extraditee of his right to apply for bail, provided that a certain standard for the grant is satisfactorily met.

An extradition proceeding being sui generis, the standard of proof required in granting or denying bail can neither be the proof
beyond reasonable doubt in criminal cases nor the standard of proof of preponderance of evidence in civil cases. While
administrative in character, the standard of substantial evidence used in administrative cases cannot likewise apply given the object
of extradition law which is to prevent the prospective extraditee from fleeing our jurisdiction. In his Separate Opinion in Purganan,
then Associate Justice, now Chief Justice Reynato S. Puno, proposed that a new standard which he termed "clear and convincing
evidence" should be used in granting bail in extradition cases. According to him, this standard should be lower than proof
beyond reasonable doubt but higher than preponderance of evidence. The potential extraditee must prove by "clear and convincing
evidence" that he is not a flight risk and will abide with all the orders and processes of the extradition court.
In this case, there is no showing that private respondent presented evidence to show that he is not a flight risk. Consequently, this
case should be remanded to the trial court to determine whether private respondent may be granted bail on the basis of "clear and
convincing evidence."
WHEREFORE, we DISMISS the petition. This case is REMANDED to the trial court to determine whether private respondent is
entitled to bail on the basis of "clear and convincing evidence." If not, the trial court should order the cancellation of his bail bond and
his immediate detention; and thereafter, conduct the extradition proceedings with dispatch.

Basco vs Rapatalo [A.M. No. RTJ-96-1335. March 5, 1997

Facts:

The complaint dated August 14, 1995, complainant Inocencio Basco charged respondent Judge Leo M. Rapatalo of RTC,
Branch 32, Agoo, La Union with gross ignorance or willful disregard of established rule of law for granting bail to an accused in a
murder case (Criminal Case No. 2927) without receiving evidence and conducting a hearing. Complainant, who is the father of the
victim, alleged that an information for murder was filed against a certain Roger Morente, one of three accused. The accused
Morente filed a petition for bail. The hearing for said petition was set for May 31, 1995 by petitioner but was not heard since the
respondent Judge was then on leave. It was reset to June 8, 1995 but on said date, respondent Judge reset it to June 22, 1995. The
hearing for June 22, 1995, however, did not materialize. Instead, the accused was arraigned and trial was set. Again, the petition for
bail was not heard on said date as the prosecution's witnesses in connection with said petition were not notified. Another attempt
was made to reset the hearing to July 17, 1995.
In the meantime, complainant allegedly saw the accused in Rosario, La Union on July 3, 1995. He later learned that the
accused was out on bail despite the fact that the petition had not been heard at all. Upon investigation, complainant discovered that
bail had been granted and a release order dated June 29, 1995 [1] was issued on the basis of a marginal note [2] dated June 22, 1995,
at the bottom of the bail petition by Assistant Prosecutor Manuel Oliva which stated: "No objection: P80,000.00," signed and
approved by the assistant prosecutor and eventually by respondent Judge. Note that there was already a release order dated June
29, 1995 on the basis of the marginal note of the Assistant Prosecutor dated June 22, 1995 (when the hearing of the petition for bail
was aborted and instead arraignment took place) when another hearing was scheduled for July 17, 1995.
Respondent Judge alleged that he granted the petition based on the prosecutor's option not to oppose the petition as well as
the latter's recommendation setting the bailbond in the amount of P80,000.00. He averred that when the prosecution chose not to
oppose the petition for bail, he had the discretion on whether to approve it or not. He further declared that when he approved the
petition, he had a right to presume that the prosecutor knew what he was doing since he was more familiar with the case, having
conducted the preliminary investigation. Furthermore, the private prosecutor was not around at the time the public prosecutor
recommended bail. Respondent Judge stated that in any case, the bailbond posted by accused was cancelled and a warrant for his
arrest was issued on account of complainant's motion for reconsideration. The Assistant Provincial Prosecutor apparently conformed
to and approved the motion for reconsideration.[3] To date, accused is confined at the La Union Provincial Jail.

Issue:
Is the judge allowed to grant bail without conducting a hearing to afford the prosecution the opportunity to present evidence
that the guilt of the accused is strong?

Held:
Petition is granted. Respondent Judge was REPRIMANDED with the WARNING .

It is in view of the abovementioned practical function of bail that it is not a matter of right in cases where the person is charged
with a capital offense punishable by death, reclusion perpetua or life imprisonment.
Article 114, section 7 of the Rules of Court, as amended, states,
Xxx
"No person charged with a capital offense, or an offense punishable by reclusion perpetua or life imprisonment when the
evidence of guilt is strong, shall be admitted to bail regardless of the stage of the criminal action."
Xxx

When the grant of bail is discretionary, the prosecution has the burden of showing that the evidence of guilt against the
accused is strong. However, the determination of whether or not the evidence of guilt is strong, being a matter of judicial discretion,
remains with the judge. "This discretion by the very nature of things, may rightly be exercised only after the evidence is submitted to
the court at the hearing. Since the discretion is directed to the weight of the evidence and since evidence cannot properly be
weighed if not duly exhibited or produced before the court,[7] it is obvious that a proper exercise of judicial discretion requires that the
evidence of guilt be submitted to the court, the petitioner having the right of cross examination and to introduce his own evidence in
rebuttal.

Topic: Sec 8, Rule 114 Importance of Hearing

JOCELYN V. GRAGEDA, vs. JUDGE NIETO T. TRESVALLES


[A.M. MTJ No. 04-1526 : February 02, 2004
FACTS:
The instant administrative case arose when Jocelyn V. Grageda filed an Affidavit-Complaint dated January 18, 2000 charging Judge
Nieto T. Tresvalles, Municipal Trial Court, Virac, Catanduanes, with gross ignorance of the law and abuse of authority relative to
Criminal Case No. 5307 entitled People v. Bernardo Tablizo, Jr. for murder.
The respondent judge granted bail to the accused in Criminal Case No. 5307 without the requisite bail hearing, despite the fact that
there was an eyewitness to the murder who made a positive identification of the accused. The complainant also alleged that the
amount of P30,000.00 printed on the Warrant of Arrest issued by the respondent judge appeared to be snowpaked, an indication
that another entry was previously made, possibly a no bail recommendation.
The respondent admitted that no bail hearing was conducted in Criminal Case No. 5307, but reasoned that the evidence of the guilt
of the accused was not strong. According to the respondent, the matter of granting bail is an exercise of judgment, and that the
accused should not be denied his constitutional right to bail.
ISSUE:
Whether or not the respondent judge committed a grave abuse of discretion.
RULING:
We agree that the respondent judge is administratively liable for granting bail to an accused charged with murder without conducting
the requisite bail hearing.
The importance of a hearing in applications for bail should once more be emphasized. Section 8, Rule 114 provides as follows:
Sec. 8. Burden of proof in bail application. - At the hearing of an application for bail filed by a person who is in custody for the
commission of an offense punishable by death, reclusion perpetua, or life imprisonment, the prosecution has the burden of showing
that the evidence of guilt is strong. The evidence presented during the bail hearing shall be considered automatically reproduced at
the trial but, upon motion of either party, the court may recall any witness for additional examination unless the latter is dead, or
otherwise, unable to testify.
The importance of the Rule lies on the fact that on the result of the bail hearing depends the right of an accused to provisional liberty
vis--vis the duty of the State to protect the people against dangerous elements. The resolution of the issue affects important norms
in our society: liberty on one hand, and order on the other. To minimize, if not eliminate, error and arbitrariness in a judges decision,
the Rules require the judge to hear the parties and then make an intelligent assessment of their evidence.
The respondents argument that a hearing is only necessary if there is an application for admission to bail is erroneous. As found by
the Executive Judge:
[T]he fact that the accused has not even filed yet any application for bail at the time bail was fixed on December 5, 2000 aggravates
matters. To state the obvious, there was no occasion for the respondent Judge to exercise any discretion on the matter of bail at that
point in time as the accused was not asking to be released on temporary liberty. The respondent Judge should have followed the
straight and trodden path, well-traveled by members of the bench, that bail should not be allowed in cases of murder. It might also
be worth mentioning, in passing, that the right to bail may be waived considering its personal nature. It arises from the time one is
placed in the custody of the law. The fact that the respondent Judge already granted bail when the accused has not been arrested
yet compounds the aggravation.
Admission to bail presupposes the exercise thereof in accordance with law and guided by the applicable legal principles. The
prosecution must first be accorded an opportunity to present evidence because by the very nature of deciding applications for bail, it
is on the basis of such evidence that judicial discretion is weighed against in determining whether the guilt of the accused is strong.
In other words, discretion must be exercised regularly, legally, and within the confines of due process, that is, after the evaluation of
the evidence submitted by the prosecution. In this case, the respondent judge motu proprio granted bail to the accused. The
prosecution was not even afforded an opportunity to present its evidence, in accordance with the Rules.
We likewise agree with the finding of the Executive Judge that the respondents claim of good faith will not exonerate him from
administrative liability.
The respondent Judge also argues in his Position Paper submitted to the undersigned Executive Judge that there is absolutely no
evidence to show that he was motivated by bad faith, fraud, dishonesty or corruption in granting bail. As such, he argues that his act
which was done in his official capacity is not subject to disciplinary action. Unfortunately for the respondent Judge, it is already
settled that when a judge grants bail to a person charged with a capital offense, or an offense punishable by reclusion perpetua or
life imprisonment without conducting the required [bail] hearing, he is considered guilty of ignorance or incompetence the gravity of
which cannot be excused by a claim of good faith or excusable negligence.
Section 26 cases:
RULE 114 BAIL
SEC. 26. Bail not a bar to objections on illegal arrest, lack of or irregular preliminary investigation. An application for or admission
to bail shall not bar the accused from challenging the validity of his arrest or the legality of the warrant issued therefore, or from
assailing the regularity or questioning the absence of a preliminary investigation of the charge against him, provided that he raises
them before entering his plea. The court shall resolve the matter as early as practicable but not later than the start of the trial of the
case. (n)
PEOPLE V. EJANDRA
G.R. No. 134203, May 27, 2004
FACTS OF THE CASE:

On or about July 2, 1997, in Quezon City, Elvie Ejandra, Magdalena Calunod, Edwin Tampos and Roel Revilla , while confederating,
conniving, conspiring, and helping each and one another, did then and there with the use of force, threat and intimidation, take and
carry away, a nine-year-old minor child, Ed Henderson Tan, against the will and consent of the latter nor any of his natural and legal
parents or guardian, to an unknown destination, detain, hold and control Ed Henderson Tan depriving him of his liberty, and during
their control and custody of Ed Henderson Tan, call, demand and negotiate the payment of ransom money from Eddie Tan, the
father of Ed Henderson Tan, for the safe release and return of the victim Ed Henderson, otherwise, the victim would be harmed or
killed, the victims father Eddie Tan actually paid the accused the amount of P548,000.00 as ransom money, for the safe release of
the victim to the damage and prejudice of the victim Ed Henderson Tan and his father Eddie Tan.
This is a review on automatic appeal of the Decision of the Quezon City Regional Trial Court, Branch 219, convicting the abovenamed accused of kidnapping for ransom, and sentencing them to suffer the death penalty.
In an appeal to the Supreme Court, Ejandra and Calunod asserted that they were arrested without any warrant therefor.
ISSUE:
Whether or not the accused can still challenge the validity of the arrest upon appeal of the case to the Supreme Court.
RULING OF THE COURT:
NO. The Office of the Solicitor General submits that in failing to assail any irregularity in their arrest before they were arraigned for
the crime charged, the appellants thereby waived their right to do so. The appellants even failed to file their respective ucounteraffidavits during the preliminary investigation of the charge against them at the Department of Justice. Moreover, the prosecution
adduced overwhelming evidence to prove the crime charged that the appellants were the perpetrators of the said crime.
We agree with the Office of the Solicitor General that the appellants Ejandra and Calunod waived any irregularities relating to their
warrantless arrest when they failed to file a motion to quash the Information on that ground, or to object to any irregularity in their
arrest before they were arraigned. They are now estopped from questioning the legality of their arrest.

Yusop vs Sandiganbayan
Facts:
Acting on an Affidavit-Complaint3 filed by a certain Erlinda Fadri, the Office of the Ombudsman-Mindanao issued an Order 4 dated
September 19, 1995, naming the following as respondents: benjamin Arao, Fredireck Winters, Pelaez Pantaran, Eduardo Dablo,
Efren Sissay and the city jail warden of Pagadian City. The Order also required respondents, within ten days from receipt thereof, to
submit their counter-affidavits and other pieces of controverting evidence.
The Office of the Ombudsman for Mindanao issued a Resolution dated January 15, 1998, 5 recommending the prosecution of "the
aforenamed respondents" for violation of Article 269 of the Revised Penal Code and Section 3-a in relation to Section 3-e of
Republic Act No. 3019 as amended. Significantly, the name of Petitioner Alvarez A. Yusop was included as one of the persons to be
prosecuted, although he was not one of the original respondents mentioned in the Order of September 19, 1995. Ombudsman
Aniano A. Desierto approved the recommendation. Accordingly, two Informations were filed with the Sandiganbayan. They were
docketed as Criminal Case Nos. 24524 (violation of Section 3-a of RA 3019) and 24525 (unlawful arrest under Article 269 of the
Revised Penal Code).
On April 16, 1998, an Order of Arrest was issued by the Sandiganbayan in Criminal Case No. 24524. Petitioner, however, posted a
bail bond before the Regional Trial Court of Dipolog City on May 20 of the same year. On the same day, he filed a "Motion To
Remand Case To The Ombudsman - Mindanao For Preliminary Investigation."
In Resolution dated June 8, 1998, the Sandiganbayan denied the Motion of petitioner for his alleged failure to submit himself to the
jurisdiction of the anti-graft court.
On August 8, 1998, petitioner filed a Motion to Dismiss, grounded again on the lack of preliminary investigation. In an Order dated
September 22, 1998, the Sandiganbayan resolved not to take action on the Motion, because petitioner had not yet submitted
himself to its jurisdiction insofar as Criminal Case No. 24525 was concerned.
On the scheduled arraignment on February 15, 1999, petitioner reiterated his claim that he had not been accorded preliminary
investigation. In its two assailed Orders, the Sandigabayan rejected his claim and proceeded with the arraignment.
Hence, this recourse.6
Issue :
1.

Whether the Sandiganbayan, despite being informed of the lack of preliminary investigation with respect to petitioner, In
Criminal Case No. 24524, committed grave abuse of discretion in proceeding with his arraignment?

2.

Is the filing of the bail bond constitute a waiver for seeking the right of preliminary investigation?

Held: Petition is partly meritorious remanded in Sandiganbayan for conduct of Preliminary investigation.
Annent to the first issue. We disagree with the Sandiganbayan's reliance on Section 27 of Republic Act 6770. 18 This provision
cannot justify the evasion of the requirement set forth in the Rules of Court for conducting preliminary investigation. The law does
not sanction such interpretation, for it deals merely with the finality of orders, directives and decisions of the Office of the
Ombudsman -- not the deprivation of the substantive right to a preliminary investigation. Moreover, petitioner cannot be bound by
the Ombudsman's January 15, 1998 Resolution, which recommended the filing of charges. He was not a party to the case and was
not accorded any right to present evidence on his behalf.

In any event, even the Ombudsman agrees that petitioner was deprived of this right and believes that the basic rudiments of due
process are complied with."19 For its part, the Sandiganbayan opted to remain silent when asked by this Court to comment on the
Petition.
Anent to the second issue:
In Go v. Court of Appeals,16 this Court held that "the right to preliminary investigation is waived when the accused fails to invoke it
before or at the time of entering a plea at arraignment." Conversely, if the accused does invoke it before arraignment, as the
petitioner did in this case, the right is not waived.
Neither did the filing of a bail bond constitute a waiver of petitioner's right to preliminary investigation. Under
Section 26, Rule 114 of the Revised Rules of Criminal Procedure,
"[a]n application for or admission to bail; shall not bar the accused from challenging the validity of his arrest or the legality of the
warrant issued therefor, or from assailing the regularity or questioning the absence of a preliminary investigation of the charge
against him, provided that he raises them before entering his plea. xxx."
We stress that the right to preliminary investigation is substantive, not merely formal or technical. To deny it to petitioner would
deprive him of the full measure of his right to due process.17 Hence, preliminary investigation with regard to him must be conducted.

RULE 116
ARRAIGNMENT AND PLEA
PEOPLE OF THE PHILIPPINES, plaintiff-appellee
vs.
EDUARDO AGBAYANI y MENDOZA, accused-appellant.
[G.R. No. 122770. January 16, 1998]

FACTS OF THE CASE:

Before this Court on automatic review is the decision of the Regional Trial Court of Quezon City, Branch 106, in view of the death
penalty imposed by it for the crime of rape, defined and penalized under Article 335 of the Revised Penal Code, as amended by
R.A. 7659.

On 12 September 1994, the Station Investigation and Intelligence Division of the National Capital Region Command,
Philippine National Police (PNP), endorsed to the Office of the City Prosecutor of Quezon City the complaint of Eden Agbayani
(hereafter EDEN) for rape against her father, herein accused-appellant Eduardo Agbayani y. Mendoza.

After appropriate preliminary investigation, a complaint for rape signed by EDEN, assisted by her sister Fedelina
Agbayani, and subscribed and sworn to before Asst. City Prosecutor Charito B. Gonzales, was filed against appellant with the
Regional Trial Court of Quezon City on 27 October, 1994. The case was docketed as Criminal Case No. Q-94-59149, then set for
arraignment, pre-trial and trial on 22 December 1994.

At his arraignment on 22 December 1994, appellant, assisted by Attys. Samuel Baldado and Edwin dela Cruz as counsel
de oficio, entered a plea of not guilty. Upon agreement of the parties, trial on the merits immediately followed, with the prosecution
presenting the first witness, Dr. Florante Baltazar, a Medico-Legal Officer of the PNP Crime Laboratory, who cross-examined by Atty.
Baldado. On the succeeding dates of trial, the prosecution presented EDEN and SPO1 Salvador Buenviaje. During these hearings,
however, appellant was represented by Atty. Arturo Temanil of the Public Attorneys Office.

After the trial proper, the trial court convict the victim and sentence him to death penalty, this record is forwarded to the
higher court for automatic review of the case.

ISSUE:
Whether or not the petitioner was apprise of his right to have counsel of his own choice.

RULING OF THE COURT:

Has been held to be mandatory and denial of this right is reversible error and a ground for new trial.
The courts should comply with Rule 116, Sec. 3. It would be a grievous error to proceed by sentencing the accused
without due process of law and this is not complete, when the accused is denied the right recognized by said rule. The records
must show compliance therewith or that the accused renounced his right to be assisted by counsel. These is demanded by the
interest of justice and remove all doubt that if the accused had waived said right, he was fully informed before giving his plea of its
consequences. Omission by courts whether voluntary should not truly be censured but also condemned.
Since appellant has miserably failed to show that he was not informed of his right to counsel, the presumptions that the
law has been obeyed and official duty has been regularly performed by the trial court stand. In other words, the trial court is
presumed to have complied with its four-fold duties under Section 6 of Rule 116 of the Rules of Court, namely,
(1) to inform the accused that he has the right to have his own counsel before being arraigned;
(2) after giving such information, to ask accused whether he desires the aid of counsel;
(3) if he so desires to procure the services of counsel, the court must grant him reasonable time to do so; and
(4) if he so desires to have counsel but is unable to employ one, the court must assign counsel de oficio to defend him.

It is settled that the failure of the record to disclose affirmatively that the trial judge advised the accused of his right to
counsel is not sufficient ground to reverse conviction. The reason being that the trial court must be presumed to have complied with
the procedure prescribed by law for the hearing and trial of cases, and that such a presumption can only be overcome by an
affirmative showing to the contrary. Thus it has been held that unless the contrary appears in the record, or that it is positively
proved that the trial court failed to inform the accused of his right to counsel, it will be presumed that the accused was informed by
the court of such right.
WHEREFORE, judgment is hereby rendered AFFIRMING the decision of the Regional Trial Court of Quezon City, Branch
106, in Criminal Case No. Q-94-59149 finding accused-appellant EDUARDO AGBAYANI y MENDOZA guilty beyond reasonable
doubt as principal of the crime of rape defined and penalized under under Article 335 of the Revised Penal Code, as amended by
R.A. No. 7659, and imposing upon him the penalty of DEATH, subject to the above modification as to the amount of indemnity.
Upon the finality of this Decision, let certified true copies thereof, as well as the records of this case; be forwarded without
delay to the Office of the President for possible exercise of executive clemency pursuant to Article 83 of the Revised Penal Code, as
amended by Section 25 of R.A. No. 7659.
With costs de oficio.
SO ORDERED.

PEOPLE OF THE PHILIPPINES, appellee, vs. WILLIAM ONG y LI and CHING DE MING @ ROBERT TIU, appellants.
(G.R. No. 137348

June 21, 2004)

Facts:
On July 27, 1998 accused William Ong y Li and Ching De Ming @ Robert Tiu, both Chinese nationals, were charged with
violation of Section 15, Article III, in relation to Section 2, Article I, of Republic Act No. 6425, otherwise known as The Dangerous
Drugs Act of 1972, as amended. TheInformation states that the two accused, on July 24, 1998 were busted in an entrapment
operation selling or offering for sale for 980.50 grams of shabu
Upon arraignment, the two 2 accused pled not guilty. The records do not show whether they had sufficient
knowledge of the English language. Their trial proceeded. In the course of the trial, the two 2 accused were given the services of
a Chinese interpreter.
On November 1998 the trial court convicted the Ong and De Ming and imposed on them the penalty of death. They each
were also ordered to pay a fine of P1 million pesos. The 2 accused on the other hand maintained their innocence and averred that
there guilt was not proven beyond reasonable doubt.
The case was brought to the SC on automatic review.
Issue: Was the crime allegedly committed by appellants duly proven and thus warrant the penalty of
death?
Ruling:
Rule 116, Section 1 (a) of the Revised Rules of Criminal Procedure, as amended, provides:
SEC. 1. Arraignment and plea; how made.(a) The accused must be arraigned before the court where the complaint or information was filed or assigned for trial. The
arraignment shall be made in open court by the judge or clerk by furnishing the accused with a copy of the complaint or
information, reading the same in the language or dialect known to him, and asking him whether he pleads guilty or not guilty.
The prosecution may call at the trial witnesses other than those named in the complaint or information.

The arraignment of appellants violates the above rule. Appellants are Chinese nationals. Their Certificate of
Arraignment states that they were informed of the accusations against them. It does not, however, indicate whether the
Information was read in the language or dialect known to them.
What leaps from the records of the case is the inability of appellants to fully or sufficiently comprehend any other
language than Chinese and any of its dialect. Despite this inability, however, the appellants were arraigned on an
Information written in the English language.
The requirement that the information should be read in a language or dialect known to the accused is mandatory. It must be strictly
complied with as it is intended to protect the constitutional right of the accused to be informed of the nature and cause of the
accusation against him. The constitutional protection is part of due process. Failure to observe the rules necessarily nullifies the
arraignment.
More important than the invalid arraignment of the appellants, the evidence by the prosecution failed to prove that
appellants willfully and unlawfully sold or offered to sell shabu.
Appellants conviction is based on the lone testimony of SPO1 Gonzales. He was the designated poseur-buyer in the team formed
for the buy-bust operation. But records reveal that he was not privy to the sale transaction that transpired between the CI and
appellant William Ong, the alleged pusher.
Also the claim of entrapment by the police officers failed to convince that indeed there was a transaction that took place. The
prosecution evidence about the buy-bust operation is incomplete. The confidential informant who had sole knowledge of how the
alleged illegal sale of shabu started and how it was perfected was not presented as a witness. His testimony was given instead by
SPO1 Gonzales who had no personal knowledge of the same.
WHEREFORE, the Decision of the court a quo is REVERSED and SET ASIDE. Appellants WILLIAM ONG y LI and CHING DE
MING @ ROBERT TIU, are ACQUITTED
G.R. No. 106695 August 4, 1994
EDWARD T. MARCELO, DIONILO D. MARFIL, CELIA C. CABURNAY, and DANIEL T. PASCUAL, petitioners,
vs.
THE COURT OF APPEALS, THE PEOPLE OF THE PHILIPPINES, HON. PEDRO T. SANTIAGO, in his capacity as The
Presiding Judge of the Regional Trial Court of Quezon City, Branch 101, and THE QUEZON CITY PROSECUTOR,
respondents.
FACTS:
In a complaint-affidavit sworn to on 18 March 1991 and filed with the Office of the City Prosecutor of Quezon City, Jose T. Marcelo
charged the petitioners with falsification of public documents committed by forging the signature of Jose P. Marcelo, Sr. in six voting
trust agreements (VTA's). Submitted in support of the affidavit-complaint were the findings of the National Bureau of Investigation
(NBI) and of the PC/PNP Crime Laboratory that the signature on the VTA's purporting to be that of Jose P. Marcelo, Sr. and the
specimen or standard signature of the latter were not written by one and the same person. After conducting a preliminary
investigation, Assistant City Prosecutor Domingo Israel found "more than sufficient evidence" of the forgery of the signature of Jose
P. Marcelo, Sr., as "found and concluded by two (2) national police agencies, the NBI and PCCL," and recommended the filing of the
case in court.
On 30 May 1991, an information for falsification of public documents was filed with the Regional Trial Court (RTC) of Quezon City,
docketed as Criminal Case No. Q-91-21285, and raffled to Branch 96 5 thereof (hereinafter Bersamin court).
On 13 June 1991, the petitioners, in a Manifestation and Motion, informed the Bersamin court of the filing of their Motion for Review
and prayed that further proceedings in the case be suspended until the resolution of the Motion for Review. Then, on 27 August
1991, the petitioners filed an urgent motion to defer the arraignment on 28 August 1991 until the resolution of their Motion for
Review. 12 Acting thereon, and over the vigorous opposition of the private prosecutor, Judge Bersamin issued an order on 28 August
1991 13 resetting the arraignment to 8 October 1991 and directing the City Prosecutor of Quezon City "to conclude the pending
review of the resolution of the filing Prosecutor Domingo Israel and to render a report of the results of the review on or before" 8
October 1991.
On 15 November 1991, the Review Committee handed down a resolution, 16 approved by Acting City Prosecutor Lydia Navarro on
29 November 1991, recommending the reversal of the Israel resolution and the withdrawal of the information in Criminal Case No.
Q-91-21285. Then on 5 December 1991, the petitioners filed a Manifestation and Motion informing the Bersamin court of the
reversal and praying for the dismissal of the case. 17 This was followed on 10 December 1991 by the motion of Assistant City
Prosecutor Conrado M. Jamolin which prayed for the withdrawal of the information in Criminal Case No. Q-91-21285 because of the
resolution of the review committee. 18 The private prosecutor opposed this motion.
On 13 December 1991, Judge Bersamin, agreeing with the findings and conclusions of the Review Committee, issued an order
granting the motion to dismiss of the accused and motion to withdraw information of the public prosecutor. However, on 27 January
1992, then Secretary of Justice Silvestre R. Bello III handed down a resolution granting the complainant's appeal, reversing the 15
November 1991 Resolution of the Review Committee, and ordering the filing of a new information.
The new information, 23 signed by Assistant City Prosecutor Ralph Lee, was filed on 5 February 1992 pursuant to the resolution of
Secretary Bello, docketed as Criminal Case No. Q-92-28104, and then raffled to Branch 101 presided over by Judge Pedro
Santiago (hereinafter Santiago court) of the RTC of Quezon City. The petitioners posted bail.
On 3 March 1992, the petitioners filed a Motion to Quash the Information on the ground that the dismissal of Criminal Case No. Q91-21285 was already final and that the appeal subsequently taken by the private prosecutor to and the resolution thereon by the
Secretary of Justice are null and void and cannot be a valid basis for any authority to file the new information or for the court to
acquire jurisdiction over the case. On 20 March 1992, Judge Santiago issued an order denying the motion to quash on the principal
ground that it was not based on any of the grounds enumerated in Section 3, Rule 117 of the Rules of Court.

ISSUE:

Whether a pre-arraignment dismissal of a criminal case by the trial court, which relied on the reversal by the Review Committee of
the Office of the City Prosecutor of the investigating prosecutor's resolution to file the information, bars the filing of a new information
for the same offense after the Secretary of Justice reversed the resolution of the review committee.
RULING:
The petition is without merit.
The Court of Appeals correctly dismissed the petitioners' special civil action for certiorari not necessarily for the reason it relied
upon, i.e., "certiorari and prohibition are not the correct remedies against an order denying a motion to quash," but because the
Santiago court did not act without or in excess of jurisdiction or with grave abuse of discretion in denying the motion to quash. It is
settled that if a court, in denying the motion to quash (or a motion to dismiss), acts without or in excess of jurisdiction or with grave
abuse of discretion, certiorari or prohibition lies. The denial by the Santiago court of the motion to quash suffers from no fatal
infirmity.
The petitioners' contention that the prosecutor did not have the authority to file the information because he acted upon an order of
the Secretary of Justice which is void in the light of Crespo vs. Mogul 37 is untenable. Nothing in the said ruling forecloses the power
or authority of the Secretary of Justice to review resolutions of his subordinates in criminal cases. The Secretary of Justice is only
enjoined to refrain as far as practicable from entertaining a petition for review or appeal from the action of the prosecutor once a
complaint or information is filed in court. In any case, the grant of a motion to dismiss, which the prosecution may file after the
Secretary of Justice reverses an appealed resolution, is subject to the discretion of the court.
Pursuant to Section 1 of Department Circular No. 7, only resolutions of the Chief State Prosecutor, the Regional State Prosecutor,
and the Provincial or City Prosecutor dismissing a criminal complaint may be appealed to the Secretary of Justice, except as
otherwise provided in Section 4 thereof. Under the latter, a resolution of the aforesaid prosecutors finding probable cause may be
appealed only upon a showing of manifest error or grave abuse of discretion; however, even with such showing, the appeal shall not
be entertained if the appellant had already been arraigned, and if the arraignment took place during the pendency of the appeal, the
appeal shall be dismissed motu proprio by the Secretary of Justice. In this case, the petitioners did not at once appeal to the
Secretary of Justice from the resolution of Assistant Prosecutor Israel. Instead, they initially filed the Motion for Review.
It is clear that the Bersamin court knew and took cognizance of the Motion for Review, deferred the arraignment of the accused until
the resolution of the said motion, and even directed the Office of the City Prosecutor "to conclude the pending review . . . and to
render a report of the results of the review. In thus recognizing and allowing the Motion for Review, the Bersamin court deferred to
the authority of the prosecution arm of the government to resolve with finality the issue of whether or not the information should have
been filed. The Review Committee's resolution was of course not final because under Department Circular No. 7 both the offended
party and the petitioners could still appeal therefrom to the Secretary of Justice under Section 1 and Section 4 thereof. The
Bersamin court knew or was expected to know, since it had to take judicial notice of Department Circular No. 7, that the resolution of
the Review Committee was not final. The offended party had, in fact, appealed from the said resolution to the Secretary of Justice
on 10 December 1991.
Consequently, the 5 December 1991 Manifestation and Motion of the petitioners praying for the dismissal of the case and the 10
December 1991 motion of Assistant City Prosecutor Jamolin asking for the withdrawal of the information were prematurely filed,
because as to the first, the period of the offended party to appeal from the resolution to the Secretary of Justice had not yet lapsed
or even begun, there being no showing of the date the offended party received a copy thereof; and, as to the second, an appeal had
in fact been filed on 10 December 1991.
The withdrawal of the information in Criminal Case No. Q-91- 21285, or even the dismissal of the said case as decreed by the
Bersamin court, did not bar the filing of a new information as directed by the Secretary of Justice in his Resolution of 27 January
1992. No jeopardy had attached as a result of the earlier termination of Criminal Case No. Q-91-21285 because the petitioners
therein had not been arraigned and had, in fact, asked for its dismissal for a cause other than that which would constitute double
jeopardy.The Santiago court, therefore, correctly denied the petitioners' motion to quash in Criminal Case No. Q-92-28104 and the
Court of Appeals committed no reversible error in dismissing the petition in CA-G.R. SP No. 27681.
G.R. No. 81389 February 21, 1989
PEOPLE OF THE PHILIPPINES, petitioner,
vs.
HON. RENATO C. DACUDAO, Presiding respondents.
FACTS :
On August 11, 1987, an Information for Murder with the qualifying circumstances of treachery and evident premeditation
was filed before the Regional Trial Court of Cebu, Branch XIV, presided by respondent Judge Renato C. Dacudao, against accused
Rey Christopher Paclibar and Nero Desamparado for the death of Cesarlito Nolasco. Upon arraignment, accused Rey Christopher
Paclibar entered a plea of 'not guilty' to the offense charged. On September 18, 1987, accused Rey Christopher Paclibar filed a
motion for bail, furnishing the Provincial Fiscal of Cebu with a copy thereof. On September 29, 1987, and without conducting a
hearing in the application for bail, respondent Judge summarily issued the following Order: ORDER .Considering the motion for bail
and the opposition thereto, and, on the basis of the complaint at bar and the sworn statement of Patrolman Elpidio Desquitado,
Tadeo Abello and Romeo Torrizo all of the Integrated National Police, Bantayan (Cebu) Police Station, which constitute the essential
evidence (so far) of the prosecution in this case, this Court hereby resolves to grant the motion for bail presented by Atty. Bernardito
A. Florida and to this end hereby fixes the bailbond for the accused Rey Christopher Paclibar at P50,000.00. From the foregoing
Order, private prosecutor Alex R. Monteclar filed a motion for reconsideration alleging that "THE GRANTING OF BAIL TO THE
ACCUSED WITHOUT A HEARING IS VIOLATIVE OF PROCEDURAL DUE PROCESS, HENCE. NULL AND VOID
ISSUE:
Whether or not the granting of bail to the accused without a hearing is violative of Procedural Due Process.
RULING:
The answer is in the affirmative. We are of the considered opinion that whether the motion for bail of a defendant who is in
custody for a capital offense be resolved in a summary proceeding or in the course of a regular trial the prosecution must be given
an opportunity to present, within a reasonable time, all the evidence that it may desire to introduce before the court should resolve
the motion for bail. If, as in the criminal case involved in the instant special civil action, the prosecution should be denied such an

opportunity, there would be a violation of procedural due process, and the order of the court granting bail should be considered void
on that ground. Bail is not a matter of right as regards persons charged with offenses punishable by reclusion perpetua when the
evidence of guilt is strong. Thus, Sec. 5, Art. 114 of the Rules of Criminal Procedure requires a hearing before resolving a motion for
bail by persons charged with offenses punishable by reclusion perpetua where the prosecution may discharge its burden of showing
that the evidence of guilt is strong. The case at bar, which is murder, is punishable by reclusion perpetua.
Finally, the defense contends that the Judge did not commit any error because actually the complaint in the Municipal Circuit Trial
Court is for homicide only) and the recommended Information was also for homicide. We note, however, that when the same was
filed with the Regional Trial Court, it was already an Information for murder. The amendment or changing of an information prior to
the plea of the accused is allowed there being no prejudice to him. Thus, in the case of Gaspar v. Sandiganbayan (144 SCRA 415
[1986]), this Court held that, "no actual double jeopardy exists where the petitioner had not yet pleaded guilty to the offense."

You might also like